How do I simplify sigma notation to find the sum in terms of n only?

Click For Summary
To calculate the area under the curve 5x + x² over the interval [0,1] using Riemann sums, the initial formula is expressed in sigma notation. The user simplifies the notation by separating terms and applying sum formulas for positive integers, leading to an expression involving sums of k and k². After correcting a multiplication error, the final result simplifies to a limit of 17/6 as n approaches infinity. This process highlights the importance of careful manipulation of sigma notation to achieve the desired simplification.
Youngster
Messages
37
Reaction score
0

Homework Statement



I'm actually asked to calculate the area under the curve 5x + x2 over the interval [0,1] using Riemann Sums. I found the formula for the Riemann sum over the interval, it being the following:

\sum^{n}_{k=1} (\frac{5k}{n}+\frac{k^{2}}{n^{2}})(\frac{1}{n})

However, I am asked to simplify the sigma notation to find the sum in terms of n only, which is where I'm currently stuck.

Homework Equations



None, I believe. Except perhaps the sum formulas for positive integers

The Attempt at a Solution



I actually just went ahead and multiplied through and separated each term like so

\frac{5}{n^{2}}\sum^{n}_{k=1} k +\frac{1}{n^{3}}\sum^{n}_{k=1} k^{2}

I, however, was left with something completely different from the expected answers given. Am I going through this correctly?
 
Physics news on Phys.org
Yes, you are on the right track. Now use a closed-form expression for those two sums to get something only in terms of n; then you can take the limit as n goes to infinity.
 
Okay, I figured out what I was doing wrong after going through my work - I neglected to multiply part of the numerator of the first sum by 3n, thus giving me something different. So after multiplying through, the end result should be:

\frac{15n^{3}+15n^{2}}{6n^{3}} + \frac{2n^{3}+3n^{2}+n}{6n^{3}}

This should simplify to:

\frac{17n^{3}+18n^{2}+n}{6n^{3}}

This could further be simplified by removing \frac{17n^{3}}{6n^{3}} to get

\frac{17}{6} + \frac{18n+1}{6n^{2}}, where an n is distributed out of the second term.

Then the limit should be \frac{17}{6} as n approaches infinity since the second term is equal to zero. :D
 
Youngster said:
Okay, I figured out what I was doing wrong after going through my work - I neglected to multiply part of the numerator of the first sum by 3n, thus giving me something different. So after multiplying through, the end result should be:

\frac{15n^{3}+15n^{2}}{6n^{3}} + \frac{2n^{3}+3n^{2}+n}{6n^{3}}

This should simplify to:

\frac{17n^{3}+18n^{2}+n}{6n^{3}}

This could further be simplified by removing \frac{17n^{3}}{6n^{3}} to get

\frac{17}{6} + \frac{18n+1}{6n^{2}}, where an n is distributed out of the second term.

Then the limit should be \frac{17}{6} as n approaches infinity since the second term is equal to zero. :D

Sure, that's it.
 
Question: A clock's minute hand has length 4 and its hour hand has length 3. What is the distance between the tips at the moment when it is increasing most rapidly?(Putnam Exam Question) Answer: Making assumption that both the hands moves at constant angular velocities, the answer is ## \sqrt{7} .## But don't you think this assumption is somewhat doubtful and wrong?

Similar threads

  • · Replies 8 ·
Replies
8
Views
2K
  • · Replies 4 ·
Replies
4
Views
1K
  • · Replies 22 ·
Replies
22
Views
966
  • · Replies 14 ·
Replies
14
Views
2K
Replies
3
Views
3K
  • · Replies 6 ·
Replies
6
Views
2K
  • · Replies 1 ·
Replies
1
Views
2K
  • · Replies 7 ·
Replies
7
Views
1K
  • · Replies 1 ·
Replies
1
Views
1K
  • · Replies 4 ·
Replies
4
Views
3K